Find the sum. 1. -7+(-5)
O-12
O-2
0 2
0 12​

Answers

Answer 1

Answer:

-12

Step-by-step explanation:

-7+(-5)=

-7-5=

-12


Related Questions

Lena, Hong and David sent a total of 126 text messages over their cell phones during the weekend. David sent four times as many messages as Hong. sent six more messages than Lena. How many messages did they each send?

Answers

Step-by-step explanation:

Which is the first sincetist of world

PLEASE ANSWER! Which expression is equal to the length of the hypotenuse of a right triangle, formed inside the unit circle, with a radius of 1?
A: sin 0/ cos 0
B: sin^2 0 + tan^2 0
C: sin 0 + cos 0
D: sin^2 0 + cos^2 0​

Answers

Answer:

b

Step-by-step explanation:

b: sin^2 0 + tan^2 0 this is just a gut feelings its been awhile since i done this kind of think i hope i could help

The expression equal to the length of the hypotenuse of a right triangle formed inside the unit circle with a radius of 1 is Option (D) [tex]sin^{2}[/tex]θ+[tex]cos^{2}[/tex]θ

What is Right triangle?

A right-angled triangle is a type of triangle that has one of its angles equal to 90 degrees.

What is Hypotenuse?

A hypotenuse is the longest side of a right-angled triangle, the side opposite the right angle.

Here,

The length of the hypotenuse of a right triangle, formed inside the unit circle, with a radius of 1 is 1 unit.

We know that,

[tex]sin^{2}[/tex]θ+[tex]cos^{2}[/tex]θ=1

Hence, The expression equal to the length of the hypotenuse of a right triangle formed inside the unit circle with a radius of 1 is Option (D) [tex]sin^{2}[/tex]θ+[tex]cos^{2}[/tex]θ

Learn more about Right triangle and Hypotenuse here

https://brainly.com/question/2869318

#SPJ2

what is the volume of a hempisphere with a radius of 9.7 cm rounded to the nearest 10th of a cubic centimeter

Answers

Answer:

volume of hemisphere =1912.26cm^3

Step-by-step explanation:

a westward moving motorcycle slows down from 24.0 m/a to 12.0 m/s in 3.0 seconds. what is the magnitude and direction of the acceleration

Answers

Answer:

0

Step-by-step explanation:

At what point on the curve x = 6t2 + 6, y = t3 − 2 does the tangent line have slope 1 /2 ?

Answers

Answer:

Hello,

P=(30,6)

Step-by-step explanation:

[tex]x=6t^2+6\\y=t^3-2\\\\\dfrac{dx}{dt}= 12t\\\dfrac{dy}{dt}= 3t^2\\\\\dfrac{dy}{dx} =\dfrac{\dfrac{dy}{dt} }{\dfrac{dx}{dt} } =\dfrac{3t^2}{12t} =\dfrac{t}{4} \\\\\dfrac{t}{4} =\dfrac{1}{2} \Longrightarrow t=2\\\\\\x=6t^2+6=6*2^2+6=30\\\\y=t^3-2=2^2-2=8-2=6\\\\\\Tangence\ point=(30,6)\\[/tex]

The point on the curve x = 6t² + 6, y = t³ - 2 where the tangent line have slope 1/2 is (30, 6).

How to depict the point on the curve?

From the information given, x = 6t² + 6, y = t³ - 2. We'll find the first order derivative of x and y which will be:

dx/dt = 12t

dy/dt = 3t²

Therefore, 3t²/12t = t/4, t = 2.

We'll put the value of t back into the equations.

x = 6t² + 6,

x = 6(2)² + 6

x = 24 + 6 = 30

y = t³ - 2.

y = (2)³ - 2

y = 8 - 2 = 6

In conclusion, the correct options is (30, 6).

Learn more about slope on:

https://brainly.com/question/3494733

A 2-column table with 9 rows. The first column is labeled year with entries 1970, 1975, 1980, 1985, 1990, 1995, 2000, 2005, 2010. The second column is labeled pounds of trash with entries 3.25, 3.25, 3.66, 3.83, 4.57, 4.52, 4.74, 4.69, 4.44. The table shows the average number of pounds of trash generated per person per day in the United States from 1970 to 2010. Use the statistics calculator to calculate the mean and median. Round the answers to the nearest hundredth. Median = Mean =

Answers

Answer:

Median: 4.44

Mean: 4.11

On edge

Step-by-step explanation:

The mean and median of the data is

Mean ≈ 4.1 pounds

Median = 4.44

How to find mean and median of a data?

The ratio of the total number of observations to the sum of the observations is known as the mean.

The median is a value for an ordered data collection that has the same amount of observations on its left and right (in either ascending or descending order).

We have the following data:

3.25, 3.25, 3.66, 3.83, 4.57, 4.52, 4.74, 4.69, 4.44

So, Mean of the data is

= sum/number of observations

= (3.25 + 3.25 + 3.66 + 3.83 + 4.57 + 4.52 + 4.74 + 4.69 + 4.44) / 9

= 36.95 / 9

= 4.10555

Now, Arranging the data in ascending order gives

3.25, 3.25, 3.66, 3.83, 4.44, 4.52, 4.57, 4.69, 4.74

Here mid value is the 5th value from both end.

Thus, median of the data set = 4.44

Learn more about mean and median here:

brainly.com/question/16118626

#SPJ6

Match the expressions given in words with their algebraic expressions.
&+59
5(5—9)
5+39
5(9-5)
-56

Answers

Answer:

5+39 this is the answer 5+39

A box of colored crayons contains 11 distinct colors. In how many ways can 2 colors be chosen, assuming that the order of the colors chosen doesn’t matter? (If necessary consult a list of formulas.)

Answers

Answer:

55

Step-by-step explanation:

The first crayon is chosen out of 11,so there are 11 choices

11

The second crayon is chosen out of 10 crayons left so there are 10 choices

10

11*10 = 110

But we do not care about the order, so we divide by 2 since there are 2 places

110/2 =55

Answer:

55

Step-by-step explanation:

Equation is a = n * (n-1)

11 * 10 = 110

Order doesn't matter, and two colors, so divide by 2.

110/2 = 55

A package of 8-count AA batteries costs $6.40. A package of 20-count AA batteries costs $15.80. Which statement about the unit prices is true?

Answers

Answer:

The unit price of the 20 pack is $0.79 and the unit price for the 8 pack is $0.80.

Step-by-step explanation:

Simply Take the price of the pack of batteries divided by the number within the pack.

$6.40 / 8 == $0.80

$15.80 / 20 == $0.79

Cheers.

The question is incomplete. You can find the missing content below.

A package of 8-count AA batteries costs $6.40. A package of 20-count Of batteries costs $15.80. Which statement about the unit prices is true?

A) The 8-count pack of AA batteries has a lower unit price of $0.79 per battery.

B) The 20-count pack of AA batteries has a lower unit price of $0.80 per battery.

C) The 8-count pack of AA batteries has a lower unit prices of $0.80 per battery.

D) The 20-count pack of AA batteries has a lower unit price of $0.79 per battery.

The correct option is Option D: The 20-count pack of AA batteries has the lower price of $0.79 per battery.

What is inequality?

Inequality is the relation between two numbers or variables or expressions showing relationships like greater than, greater than equals to, lesser than equals to, lesser than, etc.

For example 2<9

A package of 8-count AA batteries has cost = $6.40.

cost per unit count AA batteries will be= total cost of AA batteries/ number of AA batteries

= $6.40/8= $0.8

A package of 20-count AA batteries has cost = $15.80.

cost per unit count AA batteries will be= total cost of AA batteries/ number of AA batteries

= $15.80/20= $0.79

As 0.79<0.8

cost of 20-count AA batteries <  cost of 8-count AA batteries

Therefore the correct option is Option D: The 20-count pack of AA batteries has the lower price of $0.79 per battery.

Learn more about inequality

here: https://brainly.com/question/11613554

#SPJ2

Rectangle $ABCD$ is the base of pyramid $PABCD$. If $AB = 8$, $BC = 4$, $\overline{PA}\perp \overline{AD}$, $\overline{PA}\perp \overline{AB}$, and $PB = 17$, then what is the volume of $PABCD$?

Answers

Answer:

181.33

Step-by-step explanation:

Since the base of the pyramid PABCD is a rectangle, the shape in question is a rectangular based pyramid. Volume of a rectangular based pyramid is expressed as V = 1/3 * Base Area * Height of the pyramid.

Given a rectangle ABCD with AB = 8 and BC = 4, the area of the rectangle will be equivalent to the base area of the pyramid.

Base Area = Length * Breadth

Base Area = AB * BC

Base Area = 8*4 = 32

If [tex]\overline{PA}\perp \overline{AD}\ and \ \overline{PA}\perp \overline{AB}[/tex], and PB = 17, then the height of the pyramid is PB = 17.

Volume of the pyramid = 1/3 * 32 * 17

Volume of the pyramid = 1/3 * 544

Volume of the rectangular based pyramid = 181.33

change 4 5/9 from a mixed number to an improper fraction

Answers

Step-by-step explanation:

Hello, there!!

The answer would be 41/9.

The reason for above answer is to change any mixed fraction into improper fraction we should follow a simple step:

multiply the denominator with whole number.Add the answer (after mutiplied ).

look here,

=[tex] \frac{4 \times 9 + 5}{9} [/tex]

we get 41/9.

Hope it helps...

The given fraction into the improper fraction should be [tex]\frac{41}{9}[/tex]

Given that,

The mixed number fraction is [tex]4 \frac{5}{9}[/tex]

Computation:

[tex]= 4\frac{5}{9}\\\\ = \frac{41}{9}[/tex]

Here we multiply the 9 with the 4 it gives 36 and then add 5 so that 41 arrives.

learn more about the fraction here: https://brainly.com/question/1301963?referrer=searchResults

A research report claims that 20% of all individuals use Firefox to browse the web. A software company is trying to determine if the proportion of their users who use Firefox is significantly different from 0.2. In a sample of 200 of their users, 26 users stated that they used Firefox.

Required:
Using this data, conduct the appropriate hypothesis test using a 0.01 level of significance.

Answers

Answer:

Reject H0

Step-by-step explanation:

Z  = -2.474873734

 

survey (positive) 26

n (total) 200

p 0.2

 

p /point est. 0.13

POPULATION(min) 4000

np        >= 5 ? YES 40

n(1-p)  >= 5 ? YES 160

Sample is valid  

Confidence Level - "P" values  

99% 2.58

Confidence Interval - "P" values  

(0.0686 , 0.1914 )  

Reference=0.2 Reject H0

Use the Midpoint Rule with n = 10 to approximate the length of c(t) = (5 + sin(4t), 6 + sin(7t)) for 0 ≤ t ≤ 2π. (Round your answer to two decimal places.)

Answers

Answer:

  34.43

Step-by-step explanation:

A differential of length in terms of t will be ...

  dL(t) = √(x'(t)^2 +y'(t)^2)

where ...

  x'(t) = 4cos(4t)

  y'(t) = 7cos(7t)

The length of c(t) will be the integral of this differential on the interval [0, 2π].

Dividing that interval into 10 equal pieces means each one has a width of (2π)/10 = π/5. The midpoint of pieces numbered 1 to 10 will be ...

  (π/5)(n -1/2), so the area of the piece will be ...

  sub-interval area ≈ (π/5)·dL((π/5)(n -1/2))

It is convenient to let a spreadsheet or graphing calculator do the function evaluation and summing of areas.

__

The attachment shows the curve c(t) whose length we are estimating (red), and the differential length function (blue) we are integrating. We use the function p(n) to compute the midpoint of the sub-interval. The sum of sub-interval areas is shown as 34.43.

The length of the curve is estimated to be 34.43.

Many drugs used in treating schizophrenia block the reception of dopamine by neurons. (Dopamine is a neurotransmitter, which, when released by the axons of one nerve, inhibits the firing of the next nerve.) This fact led to the idea that schizophrenia occurs when too much dopamine is produced. Suppose the following data on dopamine production were obtained.

Schizophrenics Control Group
42 33
31 27
29 18

The study described above is an example of:_______

a. an independent-samples design;
b. a paired-samples design;
c. comparing a sample mean to a known population mean;
d. unknown; more information is required in order to decide.

Answers

Answer:

a. an independent-samples design.

Step-by-step explanation:

Independent sample design is the one in which samples are selected randomly. It is the observation which is not dependent on any other value. The statistical analysis is based on the assumption that the samples are independent. The study in this scenario is not dependent on any other variable and is based on independent sample design.

Help Me With This
show work​

Answers

Answer:

1. Make a list of activities and the number of students:

Watching TV: 32

Talking on the phone: 41

Video games: 24

Reading: 15

2. Then combine the data in a bar graph as shown in the picture

Help me plz will mark brainliest

Answers

Step-by-step explanation:

Given,

the bigger triangle (∆GFH), <HGF = 90° and <GFH = 60° and GF = 4

In the 30-60-90 triangle, FH = 4×2 = 8

or you can find it like this,

cos60 = 4/FH

FH = 4/cos60 = 8

Now, I is the midpoint of FH, so, FI = 8/2 = 4

now for ∆FGI,

<GFI = 60°, GF = 4, FI = 4, it's a equilateral triangle, so we can say GI = 4

Now we need to find the height of ∆FGI, for that,

area of ∆FGI = (√3/4)×4² = 4√3 = 6.92820323..

So height (GJ) = 2×area/side = 2×4√3/4 = 2√3 = 3.46410162.. [height of ∆GFI (Because altitude is perpendicular distance from a point) = GJ, given]

now, in ∆GJI, GJ = 2√3, GI = 4 and <GJI = 90° (since itsy altitude, the angle will be a right angle, i.e. 90°)

So, JI = √{4²-(2√3)²} = 2

Area = 2×2√3/2 = 2√3 = 3.46410162.. = 3.46 (rounded to the nearest hundredth)

if 80% of 2 is 200.what is 70% of 27

Answers

Answer:

18.90

Hope i got it right

I need help on this question, can someone please answer it correctly?

Answers

Answer:the one area < with line underneath then -4

St-by-step explanation: I’m pretty sure this is correct

Answer:

[tex] \boxed{x \leqslant - 4}[/tex]

Step-by-step explanation:

[tex] \mathrm{16x - 7 \leqslant - 71}[/tex]

Move constant to Right hand side and change its sign

[tex] \mathrm{16x \leqslant - 71 + 7}[/tex]

Calculate

[tex] \mathrm{16x \leqslant - 64}[/tex]

Divide both sides of the equation by 16

[tex] \mathrm{ \frac{16x}{16} \leqslant \frac{ - 64}{16} }[/tex]

Calculate

[tex] \mathrm{x \leqslant - 4}[/tex]

Hope I helped!

Best regards!

WILL MARK BRAINIEST!!! Segment AC has two endpoints; (-2,5) and (2,-5). What are the coordinates of point B on segment AC such that the ratio of AB to BC is 5:1? Any help would be appreciated; first correct answer get brainiest and a 5 star review!

Answers

Answer:

[tex](\frac{4}{3},-\frac{10}{3})[/tex]

Step-by-step explanation:

If the extreme ends of a line segment AC are A[tex](x_1,y_1)[/tex] and C[tex](x_2,y_2)[/tex].

If a point B(x, y) divides the segment in the ratio of m : n

Then the coordinates of the point B are,

x = [tex]\frac{mx_2+nx_1}{m+n}[/tex]

y = [tex]\frac{my_2+ny_1}{m+n}[/tex]

If the ends of AC are A(-2, 5) and C(2, -5) and a point B divides it in the ratio of m : n = 5 : 1

Therefore, coordinates of this point will be,

x = [tex]\frac{5\times (2)+1(-2)}{5+1}[/tex]

  = [tex]\frac{10-2}{5+1}[/tex]

  = [tex]\frac{8}{6}[/tex]

  = [tex]\frac{4}{3}[/tex]

y = [tex]\frac{5\times (-5)+1(5)}{5+1}[/tex]

  = [tex]\frac{-25+5}{6}[/tex]

  = [tex]-\frac{20}{6}[/tex]

  = [tex]-\frac{10}{3}[/tex]

Therefore, coordinates of the point B are [tex](\frac{4}{3},-\frac{10}{3})[/tex].

You need a shelf for a small space in your house, so you make a measurement with your meter stick and head to the store. Once there, you find that the dimension of the shelves you want is given in cm. If your space measured 0.8 m, and the shelves at the store measure 30 cm, answer the following questions: 1) How many meters wide is the shelf you want to buy? 2) Will it fit in your house? yes no

Answers

Answer:

1. 0.3 m

2. yes

Step-by-step explanation:

The computation is shown below:

Given that

Measurement of space = 0.8m

measurement of the shelves = 30 cm = 0.3 m as 1 m = 100 cm

So for 30 cm it would be

= 0.30 ÷ 100

= 0.3 m

Based on the above information,

1. The number of meters wide for the shelf to buy is 0.3 m

2 And yes it is fitted in the house

Using the required conversion metrics, the width of the shelf at the stis 0.3 meters and will not fit in the house.

Given the Parameters :

Measured width = 0.8 mStore width = 30 cm

Using the appropriate metric conversion values :

100 cm = 1 m

Converting the store measurement into meters :

30 cm ÷ 100 = 0.3 meters

Hence, the shelf you want to purchase measures 0.3 meters.

Since, the measured width and the width of the shelf at the store are different, then the shelf will not fit in.

Learn more : https://brainly.com/question/16867858

80% of ______ is 1,200?

Answers

Answer:

the unknown number is 1500

Step-by-step explanation:

let "a" be the unknown number we finding so from the above question we can deduce that

(80/100)*a=1200

80a=1200*100

80a=120000

a=120000/80

a=1500

Which ordered pair is a solution to the following linear system? y = x y = –x

Answers

Answer:

(2,2) (-1,-1)

Step-by-step explanation

i think this is there answer im sorry if im wrong

You are given the sample mean and the population standard deviation. Use this information to construct the​ 90% and​ 95% confidence intervals for the population mean. Interpret the results and compare the widths of the confidence intervals. From a random sample of business​ days, the mean closing price of a certain stock was ​$. Assume the population standard deviation is ​$. The​ 90% confidence interval is ​( nothing​, nothing​). ​(Round to two decimal places as​ needed.) The​ 95% confidence interval is ​( nothing​, nothing​). ​(Round to two decimal places as​ needed.) Which interval is​ wider? Choose the correct answer below

Answers

Complete Question

The  complete question is shown on the first uploaded image

Answer:

The 90% confidence interval is  [tex][108.165 ,112.895][/tex]

The  95%  confidence interval is [tex][107.7123 ,113.3477][/tex]

The  correct option is  D

Step-by-step explanation:

From the question we are told that

    The sample size is  n =  48

     The sample  mean is  [tex]\= x = \$ 110.53[/tex]

    The standard deviation is  [tex]\sigma = \$ 9.96[/tex]

Considering first question

  Given that the confidence level is  90% then the level of significance is mathematically represented as

            [tex]\alpha = (100 - 90)\%[/tex]

           [tex]\alpha = 0.10[/tex]

The  critical value  of  [tex]\frac{\alpha }{2}[/tex] from the  normal distribution table is  

          [tex]Z_{\frac{\alpha }{2} } = 1.645[/tex]

Generally the margin of error is mathematically represented as

            [tex]E = ZZ_{ \frac{x}{y} } * \frac{\sigma}{ \sqrt{n} }[/tex]

             [tex]E = 1.645 * \frac{9.96}{ \sqrt{ 48} }[/tex]

             [tex]E = 2.365[/tex]

The  90% confidence interval is  

       [tex]\= x - E < \mu < \= x + E[/tex]

=>    [tex]110.53 - 2.365 < \mu < 110.53 + 2.365[/tex]

=>    [tex]108.165 < \mu < 112.895[/tex]

Considering second question

  Given that the confidence level is  95% then the level of significance is mathematically represented as

            [tex]\alpha = (100 - 95)\%[/tex]

           [tex]\alpha = 0.05[/tex]

The  critical value  of  [tex]\frac{\alpha }{2}[/tex] from the  normal distribution table is  

          [tex]Z_{\frac{\alpha }{2} } = 1.96[/tex]

Generally the margin of error is mathematically represented as

            [tex]E = Z_{ \frac{x}{y} } * \frac{\sigma}{ \sqrt{n} }[/tex]

             [tex]E = 1.96 * \frac{9.96}{ \sqrt{ 48} }[/tex]

             [tex]E = 2.8177[/tex]

The  95% confidence interval is  

       [tex]\= x - E < \mu < \= x + E[/tex]

=>    [tex]110.53 - 2.8177 < \mu < 110.53 + 2.8177[/tex]

=>    [tex]107.7123 < \mu < 113.3477[/tex]

The length of a rectangle is 2 inches more than its width. The area of the rectangle is equal to 1 inches less than 3 times the perimeter. Find the length and width of the rectangle.

Answers

Answer:

The length is 13, when the width is 11

Step-by-step explanation:

The width is x, when the length is x+2. The area is x(x+2)= x^2+2x.

The perimeter is (x+x+2)*2= (2x+2)*2=4x+4

THe thrre times perimeter is 3(4x+4)= 12x+12

x^2+2x= 12x+12-1

x^2+2x-12x-11=0

x^2-10x-11=0

x=-1 (not suitable); x=11 - the width

The length is 11+2=13

PLEASE HELP 40 POINTS
Express each logarithm in terms of ln 3 and ln 5.
ln 81 / 125

Answers

I’ll recommend b or c so choose answer c

The solution is 4ln3 - 3ln5, which is correct option(C).

What are the properties of logarithms?

There are four basic properties of logarithms:

logₐ(xy) = logₐx + logₐy.

logₐ(x/y) = logₐx - logₐy.

logₐ(xⁿ) = n logₐx.

logₐx = logₓa / logₓb.

What are Arithmetic operations?

Arithmetic operations can also be specified by the subtract, divide, and multiply built-in functions.

The operator that perform arithmatic operation are called arithmatic operators .

Operators which let do basic mathematical calculation

+ Addition operation : Adds values on either side of the operator.

For example 4 + 2 = 6

- Subtraction operation : Subtracts right hand operand from left hand operand.

for example 4 -2 = 2

* Multiplication operation : Multiplies values on either side of the operator

For example 4*2 = 8

/ Division operation : Divides left hand operand by right hand operand

For example 4/2 = 2

According to the problem, we will use some of the basic logarithmic properties,

Given expression,

⇒ ln81/125

⇒ ln81 - ln125   [used property : logₐ(x/y) = logₐx - logₐy]

⇒ ln3⁴ - ln5³    [ 81 =3⁴ and 125 = 5³ ]

⇒ 4ln3 - 3ln5   [used property : logₐ(xⁿ) = nlogₐx]

Hence, the solution is 4ln3 - 3ln5

Learn more logarithmic properties here:

brainly.com/question/24211708

#SPJ2

Which angle of rotation is determined by the matrix below?{1/2 -sqrt3/2 sqrt3/2 1/2] 30° 60° 120° 300°

Answers

Answer:

  60°

Step-by-step explanation:

You have the rotation matrix ...

  [tex]\left[\begin{array}{cc}\cos{\theta}&-\sin{\theta}\\\sin{\theta}&\cos{\theta}\end{array}\right]=\left[\begin{array}{cc}\dfrac{1}{2}&-\dfrac{\sqrt{3}}{2}\\\dfrac{\sqrt{3}}{2}&\dfrac{1}{2}\end{array}\right][/tex]

This tells you the angle of rotation is ...

  [tex]\tan{\theta}=\dfrac{\sin{\theta}}{\cos{\theta}}=\dfrac{\left(\dfrac{\sqrt{3}}{2}\right)}{\left(\dfrac{1}{2}\right)}=\sqrt{3}\\\\\theta=\arctan{\sqrt{3}}=60^{\circ}[/tex]

The angle of rotation is 60°.

Answer:

B----- 60

Step-by-step explanation:

alicia bought 205 pink and purple beads. she lost 3/5 of the pink beads and was given another 15 purple beads.

Answers

Answer: sorry

Step-by-step explanation:

767u6777

What is the image point of (-6,3) after a translation right 5 units and down 3 units?

Answers

answer:
(-1,0)
step-by-step explanation:
(-6,3)
right 5
(-1,3)
down 3
(-1,0)

1/3 of a shipment of books weights 28 pounds

Answers

Answer:

84 pounds

Step-by-step explanation:

If 1/3 of a book is equal to 28 pounds then 28*3 will give you your answer

In the figure below, circle O has a central ángel of 120 degrees. what is the area shaded of the circle in terms of ,r, the radius? Leave your answer in terms of pi.

Answers

Answer:

the area of a section of a circle is [tex]\frac{1}{360}[/tex]* Θ * (area of the circle)

the theta/360 tells us the amount of circle currently in question

so , the answer will be 1/3 pi r^2        [A]

Other Questions
Why are over-the-counter drugs often abused?They taste better than prescription drugs.They are legal and easy to access.They do not need to be taken daily.They are more potent than prescription drugs. What is the difference between effect and affect? pls provide with some good examples as well:) 18. If the pH of a strong acid solution is known to be 3.25, what is the [H+)?(b) what is the pOH of this solution? The sum of the cubes of 3 numbers which are in the ratio 1:2:3 is 7776. Find the numbers The owners of a landscaping business decide they need insurance to cover their trucks in case of accidents , injuries caused by flying debris from their trimmers and blowers, and property damage caused by falling tree limbs. What type of policy should the owners consider to cover all of these risk? find the area of quadrilateral ABCD Write the equation in standard form.x2 + 6 = 5x Journalize the following entries for the month:a. Materials are purchased to produce 960 units. b. Conversion costs are applied to 910 units of production. c. The cell completes 860 units, which are placed into finished goods. Para la funcin f(x) = x + 6, cul esresultado de la funcin si x=2 ? Find y when x = 4 (30 points) What methods do you think can should be used to protect patient information? do you think distance and time are relevant terms in describing motion? (07.06A) Which scenario best matches the linear relationship expressed in the equation y = 13.50x + 300? Bobby has $300 in the yearbook fund and spends $13.50 on each yearbook. Bobby has $13.50 in the yearbook fund and spends $300 on each yearbook. Bobby has $300 in the yearbook fund and earns $13.50 for each yearbook sold. Bobby has $13.50 in the yearbook fund and earns $300 for each yearbook sold. The force of gravity on an object varies directly with its mass. The constant of variation due to gravity is 32.2 feet per second squared. Which equation represents F, the force on an object due to gravity according to m, the objects mass? F = 16.1m F = F equals StartFraction 16.1 Over m squared EndFraction. F = 32.2m F = F equals StartFraction 32.2 Over m squared EndFraction. Melody's Crafts 'N ThingsAndrea works for a retail shop called Melody's Crafts 'N Things. Andrea was often quite rude to customers when they annoyed her. One morning, her supervisor asked Andrea into her office and informed her that her behavior was unacceptable. To be rude to one customer could cause several customers not to come back. Andrea's supervisor told her that retaining a customer is much cheaper than trying to get a new one. After their constructive discussion, Andrea's boss told her that she wanted her to go out on the floor that day and attempt to be more helpful to customers.Over the next few months, Andrea began to help customers learn more about the different products in the store and often explained why the customer might want to purchase one product over another. In doing so, she realized the importance of her role in the retail shop and began to truly enjoy her job. She started to think about the future of the company. She even thought that the company could do very well if it were to sell its products on eBay. When she brought this idea to her boss, her boss thought it was excellent, and asked Andrea to look further into the pros and cons of her idea.After evaluating Andrea's research, her boss decided to move forward with the idea, asking Andrea to head up this new venture for the company. Andrea then realized that retail is not just about making the sale; the customer should be the focus of all operations. Andrea now looks up to her boss, and is very glad that she was there to mentor her.Refer to Melody's Crafts 'N Things. Doing business on eBay allows an entrepreneur toa. develop a business-to-business selling model.b. create place utility.c. find out what works, fast.d. explore producer markets.e. develop form utility. A rectangular coil lies flat on a horizontal surface. A bar magnet is held above the center of the coil with its north pole pointing down. What is the direction of the induced current in the coil? Bharath has made a table of content for his document in Open Office, in which he wants to make few changes, but he is unable to make the changes. Give reason. Explain how he can make the necessary changes Rewrite the paragraph, correcting all mechanical errors.According to Dr. Jones, Prof. of marine biology at Fla. International university in miami, Famous Orcas like Keiko -the whale in the movie "Free Willy" who was successfully released back into the wild-have helped raise public awareness about the inhumane capture and treatm-ent of these sea animals. In the past 40 years, a number of organizations, including the Free Willy Keiko foundation and the humane society have taken important steps to stop the capture of Orcas in the waters surrounding the U.S. Progress to stop inhuman capture and captivity throughout the world continues please help :) Which of the following would a physical geographer study? a. primary sources c. urban areas b. secondary sources d. landforms hello :) why is the first one wrong?